Fiche de mathématiques
> >

Les nombres complexes - Exercices

Partager :

exercice 1


Mettre sous forme algébrique les nombres complexes suivants :

1) \dfrac{1+2i}{3-4i}

2) \dfrac{1}{1+\dfrac{2}{i}}

3) \dfrac{1}{(1+2i)^2}

4) \dfrac{1+i}{3-i}+\dfrac{1-i}{3+i}

5) \left(\sqrt{3}-i\right)^{11}

exercice 2


A quelle condition le produit de deux nombres complexes est-il-réel (respectivement imaginaire pur)?

exercice 3


Soit \theta\in\R\backslash 2\pi\Z \enskip \text{ , } z=\dfrac{1+\cos\theta + i\sin\theta}{1-\cos\theta-i\sin\theta}\enskip \text{ . Calculer }\enskip \mathcal{R}e(z)\text{ , }\mathcal{I}m(z)\text{ , }|z| \text{ et }\arg z

exercice 4

Soient z et z' deux complexes de module 1, tels que 1+zz'\neq 0. Montrer que \dfrac{z+z'}{1+zz'}\in\R

exercice 5

Déterminer :
1) Les racines cinquièmes de -i
2) Les racines sixièmes de \dfrac{-4}{1+i\sqrt{3}}
3) Les racines quatrièmes de 476+480i

exercice 6

1) Soit \theta\in\R. Développer \cos(5\theta) \text{ et }\sin(5\theta) comme somme de puissances de \cos(\theta) \text{ et }\sin(\theta)
2) En déduire une expression de \tan(5\theta) en fonction de puissances de \tan(\theta) pour \theta\neq (2k+1)\dfrac{\pi}{10}\enskip \text{ avec } k\in\Z.

exercice 7

Soit x\in\R. Linéariser \cos^7 x

exercice 8

Soit x un réel.
1) Montrer que, pour tout p\in\N\enskip : \enskip \cos^{2p+1}(x)=\dfrac{1}{2^{2p}}\displaystyle \sum_{k=0}^{p}{2p+1\choose k } \cos\left((2p+1-2k)x\right)

2) De même, montrer que, pour tout p\in\N\enskip : \enskip \sin^{2p+1}(x)=\dfrac{(-1)^p}{2^{2p}}\displaystyle \sum_{k=0}^{p}(-1)^k{2p+1\choose k } \sin\left((2p+1-2k)x\right)

exercice 9

Résoudre les équations d'inconnu complexe suivantes:
1) 4z^2-16z+11-12i=0
2) z^2-(1+4i)z+5(1+i)=0
3) z^8+4z^4+16=0
4) (z^2+z+1)^2+1=0

exercice 10

Soit x\in\R \enskip \text{ . Calculer } \enskip \displaystyle S=\sum_{k=0}^{n}{n\choose k} \cos (kx) \enskip \text{ et }\enskip S'=\sum_{k=0}^{n}{n\choose k} \sin (kx)

exercice 11

Soit z=\text{e}^{\frac{2i\pi}{7}}\enskip \text{ . Calculer } A=z+z^2+z^4 \text{ et } B=z^3+z^5+z^6\text{ . }

On pourra calculer AB et A+B

exercice 12

1) Soit x\in\R\text{ . } Exprimer \cos(5x) en fonction de \cos(x) .
2) En déduire que \cos\left(\dfrac{\pi}{10}\right) est racine du polynôme 16X^4-20X^2+5 .
3) En déduire la valeur de \cos^2\left(\dfrac{\pi}{10}\right)\enskip\text{ puis}\enskip  \cos\left(\dfrac{\pi}{5}\right) .

exercice 13

1) \text{ Soient } (z,z')\in\C^2 \enskip\text{ .  Montrer l'identité de la médiane : } \enskip |z+z'|^2+|z-z'|^2=2(|z|^2+|z'|^2)

2) \text{ Soit } \omega\in\C \text{ tel que } \omeaga^2=zz' \text{ . Montrer que : } |z|+|z'|=\left|\omega+\dfrac{z+z'}{2}\right|+\left|\omega-\dfrac{z+z'}{2}\right|

exercice 14

Résoudre pour z\in\C\text{ , }2\arg(z+i)\equiv \arg(z)+\arg(i)\enskip[2\pi]

exercice 15

Trouver une condition nécessaire et suffisante sur les n nombres complexes non nuls z_1,z_2,\cdots,z_n pour que \displaystyle\sum_{k=1}^{n}|z_k|=\left|\sum_{k=1}^{n}z_k\right| \enskip (n\in\N^{*}\backslash\lbrace 1\rbrace )

exercice 16

Dans le plan complexe, déterminer l'ensemble des points M d'affixe z tels que :
1) z^2-2z+7\in\R
2) \left|\dfrac{z-i}{z+2}\right|=1
3) z^2-(1+i)^2=\bar{z}^2-(1-i)^2
4) z\text{ , }z-1\text{ et }\dfrac{1}{z} aient le même module

exercice 17

Soit a\in\U
On note z_1,\cdots,z_n les racines de l'équation z^n=a \enskip (n\in\N^*)
Montrer que les points du plan complexe dont les affixes sont (1+z_1)^n,\cdots, (1+z_n)^n sont alignés

exercice 18

Soient A,B,C Trois points du plan complexe, deux à deux distincts, d'affixes respectifs a,b,c. Montrer que les propositions suivantes sont équivalentes :

1) ABC est un triangle équilatéral.

2) j ou j^2 est racine du polynôme aX^2+bX+c

3) a^2+b^2+c^2=ab+bc+ca

4) (b-a)^2+(c-b)^2+(a-c)^2=0

exercice 19

Soient r_1 la rotation de centre A d'affixe -1 et d'angle \dfrac{\pi}{3} et r_2 la rotation de centre B d'affixe j et d'angle \dfrac{2\pi}{3} .
Montrer que r_2\circ r_1 est une symétrie centrale dont on précisera l'affixe du centre.

exercice 20

Soit n\in\N\text{ tel que } n\geq 3
On définit les points A_1,\cdots, A_n les n sommets d'un polygone régulier inscrit dans un cercle de rayon R, et soit M un point quelconque de ce cercle.
Montrer que la somme des distances \displaystyle \sum_{k=1}^{n} MA_k ^{2} est indépendant de M.



exercice 1


1) \begin{matrix} \dfrac{1+2i}{3-4i} &=& \dfrac{(1+2i)(3+4i)}{9+16} &=& \dfrac{3+4i+6i-8}{25} &=&\dfrac{1}{25}(-5+10i) &=& \boxed{-\dfrac{1}{5}+\dfrac{2}{5}i}\end{matrix}

2) \begin{matrix} \dfrac{1}{1+\dfrac{2}{i}} &=& \dfrac{i}{2+i} &=& \dfrac{i(2-i)}{4+1} &=& \boxed{\dfrac{1}{5}+\dfrac{2}{5}i}\end{matrix}

3) \begin{matrix} \dfrac{1}{(1+2i)^2} &=& \dfrac{1}{1+4i-4} &=& \dfrac{-3-4i}{(-3+4i)(-3-4i)} &=&\dfrac{-3-4i}{9+16} &=& \boxed{-\dfrac{3}{25}-\dfrac{4}{25}i}\end{matrix}

4) \begin{matrix} \dfrac{1+i}{3-i}+\dfrac{1-i}{3+i} &=& \dfrac{(1+i)(3+i)+(1-i)(3-i)}{9+1} &=& \dfrac{3+4i-1+3-4i-1}{10}  &=& \boxed{\dfrac{2}{5}}\end{matrix}

5)
\begin{matrix} \left(\sqrt{3}-i\right)^{11} &=& 2^{11}\left(\dfrac{\sqrt{3}}{2}-\dfrac{1}{2}i\right)^{11}&=&2^{11}\left(\cos \left(-\dfrac{\pi}{6}\right) + i \sin \left(-\dfrac{\pi}{6}\right)\right)^{11}\\\\&=&2^{11}\text{e}^{-i\frac{11\pi}{6}}&=& 2^{11} \text{e}^{i\left(\frac{\pi}{6}-2\pi\right)}\\\\&=& 2^{11}\text{e}^{i\frac{\pi}{6}}&=&\boxed{2^{10}\sqrt{3}+2^{10}i}\end{matrix}

exercice 2


Soient z,z'\in\C \enskip : \enskip z=r\text{e}^{i\theta} \text{ et } z'=r'\text{e}^{i\theta'}  \text{ où } r,r'\in\mathbb{R}^{*}_{+} \text{ et } \theta,\theta'\in\R
On a : zz'=(r\text{e}^{i\theta})(r'\text{e}^{i\theta'})=rr'\text{e}^{i(\theta+\theta')}=rr'(\cos(\theta+\theta')+i\sin (\theta+\theta'))
Pour que le produit soit réel, il faut et il suffit que la partie imaginaire \mathcal{I}m(zz') soit nulle, ce qui correspond à \sin(\theta+\theta')=0\iff \theta+\theta'=0+k\pi \text{ avec } k\in\Z

D'où : \boxed{zz'\in\R\iff \theta+\theta'\equiv0\enskip [\pi]}

De même, pour que le produit soit imaginaire pur, il faut et il suffit que la partie réelle \mathcal{R}e(zz') soit nulle, ce qui correspond à  \cos(\theta+\theta')=0\iff\theta+\theta'=\dfrac{\pi}{2}+k\pi \text{ avec } k\in\Z

\boxed{zz'\in i\R\iff \theta+\theta'\equiv \dfrac{\pi}{2}\enskip [\pi]}

exercice 3

\begin{matrix} z&=&\dfrac{1+\cos\theta + i\sin\theta}{1-\cos\theta-i\sin\theta}&=&\dfrac{1+\text{e}^{i\theta}}{1-\text{e}^{i\theta}}\\\\&=&\dfrac{\text{e}^{i\frac{\theta}{2}}\left(\text{e}^{-i\frac{\theta}{2}}+\text{e}^{i\frac{\theta}{2}}\right)}{\text{e}^{i\frac{\theta}{2}}\left(\text{e}^{-i\frac{\theta}{2}}-\text{e}^{i\frac{\theta}{2}}\right)}&=&-\dfrac{2\cos\left(\dfrac{\theta}{2}\right)}{2i\sin\left(\dfrac{\theta}{2}\right)}\\\\&=&\dfrac{\cos\left(\dfrac{\theta}{2}\right)}{\sin\left(\dfrac{\theta}{2}\right)}\enskip i&=&\boxed{\cot\left(\dfrac{\theta}{2}\right)\enskip i }\end{matrix}

On en déduit directement que : \boxed{\mathcal{R}e(z)=0\enskip , \enskip \mathcal{I}m(z)= \cot\left(\dfrac{\theta}{2}\right)}

Trouvons le module et l'argument de z

\bullet \text{ Cas 1 : }\cot\dfrac{\theta}{2}>0 . Ce qui correspond à : \displaystyle\theta\in\bigcup_{k\in\Z} ]2k\pi,\pi+2k\pi[

Donc : \boxed{|z|=\left|\cot\left(\dfrac{\theta}{2}\right)\right|=\cot\dfrac{\theta}{2}}\enskip \text{ et }\enskip \arg(z)\equiv \arg(i)\enskip[2\pi] \iff \boxed{\arg(z)\equiv \dfrac{\pi}{2}\enskip[2\pi]}

\bullet \text{ Cas 2 : }\cot\dfrac{\theta}{2}<0. Ce qui correspond à : \displaystyle\theta\in\bigcup_{k\in\Z} ]\pi+2k\pi,2(k+1)\pi[

Donc : \boxed{|z|=\left|\cot\left(\dfrac{\theta}{2}\right)\right|=-\cot\dfrac{\theta}{2}\enskip } \text{ et }\enskip \arg(z)\equiv \arg(-i)\enskip[2\pi] \iff \boxed{\arg(z)\equiv -\dfrac{\pi}{2}\enskip[2\pi]}

\bullet \text{ Cas 3 : }\cot\dfrac{\theta}{2}=0. Ce qui correspond à : \displaystyle\theta\in\pi+2\pi\Z

Donc \boxed{|z|=0 \text{ et }z \text{ n'admet pas d'argument}}

exercice 4

On a:
\begin{matrix} \dfrac{z+z'}{1+zz'}-\overline{\left(\dfrac{z+z'}{1+zz'}\right)}&=&\dfrac{z+z'}{1+zz'}-\dfrac{\bar{z}+\bar{z'}}{1+\bar{z}\bar{z'}}&=&\dfrac{z+z'}{1+zz'}-\dfrac{\frac{1}{z}+\frac{1}{z'}}{1+\frac{1}{zz'}}& \text{(En effet, puisque } |z|=|z'|=1\text{ , alors } \bar{z}=z^{-1}\text{ et } \bar{z'}=z'^{-1} \text{ ) }\\\\&=&\dfrac{z+z'}{1+zz'}-\dfrac{z'+z}{zz'+1}&=&0\end{matrix}

Il s'ensuit que \dfrac{z+z'}{1+zz'}=\overline{\left(\dfrac{z+z'}{1+zz'}\right)}

Donc : \boxed{\dfrac{z+z'}{1+zz'}\in\R}

exercice 5

1) Les racines cinquièmes de -i sont solutions de l'équation z^5=-i
On a : z^5=-i\iff iz^5=1\iff (iz)^5=1
Les racines sont donc z_k \text{ / } k\in\lbrace 0,\cdots , 4\rbrace \text{ avec } iz_k=\text{e}^{i\frac{2k\pi}{5}}
Ce qui donne z_k=-i\text{e}^{i\frac{2k\pi}{5}}=\text{e}^{-i\frac{\pi}{2}}\text{e}^{i\frac{2k\pi}{5}}=\text{e}^{i\frac{\pi}{10}(4k-5)}\enskip \text{ avec } k\in\lbrace 0,\cdots , 4\rbrace

On en déduit les racines cinquièmes de -i : \boxed{ z_0=\text{e}^{-i\frac{\pi}{2}}=-i \enskip , \enskip z_1=\text{e}^{-i\frac{\pi}{10}}\enskip , \enskip z_2=\text{e}^{i\frac{3\pi}{10}}\enskip , \enskip z_3=\text{e}^{i\frac{7\pi}{10}}\enskip , \enskip z_4=\text{e}^{i\frac{11\pi}{10}}}


2) On a: -\dfrac{4}{1+i\sqrt{3}}=\dfrac{-4(1-i\sqrt{3})}{4}=-1+i\sqrt{3}=2\text{ e}^{i\frac{2\pi}{3}}
Les racines sixièmes de -i sont solutions de l'équation z^6=2\text{ e}^{i\frac{2\pi}{3}}\iff \left(\dfrac{1}{\sqrt[6]{2}}\text{ e}^{-i\frac{\pi}{9}}z\right)^6=1
Les racines sont donc z_k \text{ / } k\in\lbrace 0,\cdots , 5\rbrace \text{ avec } \dfrac{1}{\sqrt[6]{2}}\text{ e}^{-i\frac{\pi}{9}}z_k=\text{e}^{i\frac{2k\pi}{6}}
Ce qui donne z_k= \sqrt[6]{2}\text{ e}^{i\frac{\pi}{9}(1+3k)}\enskip \text{ avec } k\in\lbrace 0,\cdots , 5\rbrace

On en déduit les racines : \boxed{ z_0=\sqrt[6]{2}\text{e}^{i\frac{\pi}{9}} \enskip , \enskip z_1=\sqrt[6]{2}\text{e}^{i\frac{4\pi}{9}}\enskip , \enskip z_2=\sqrt[6]{2}\text{e}^{i\frac{7\pi}{9}}\enskip , \enskip z_3=\sqrt[6]{2}\text{e}^{i\frac{10\pi}{9}}\enskip , \enskip z_4=\sqrt[6]{2}\text{e}^{i\frac{13\pi}{9}}\enskip , \enskip z_5=\sqrt[6]{2}\text{e}^{i\frac{16\pi}{9}}}


3)
Rappel
(x+iy)^2=a+ib \text{ ssi } \left\lbrace\begin{matrix} &x^2+y^2=\sqrt{a^2+b^2} \\ &x^2-y^2=a \\&xy=\dfrac{b}{2}\end{matrix}


Trouvons z=x+iy \text{ / } z^2=476+480i
On a :
\left\lbrace \begin{matrix} x^2+y^2=\sqrt{476^2+480^2}=676 \\x^2-y^2=476 \\ xy=\dfrac{480}{2}=240\geq 0\end{matrix}

On obtient : x^2=576 \text{ et }y^2=100 \text{ avec }xy\geq 0 \Longrightarrow (x=24 \text{ et } y=10 ) \text{ ou } (x=-24 \text{ et } y=-10)
Les racines carrées de 476+480i sont donc 24+10i et -24-10i

Cherchons de la même manière les racines carrées de chacune d'elles :

Les racines carrées de 24+10i

Posons z=x+iy \text{ / } z^2=24+10i
On obtient : x^2=25 \text{ et }y^2=1 \text{ avec }xy\geq 0 \Longrightarrow (x=5 \text{ et } y=1 ) \text{ ou } (x=-5 \text{ et } y=-1)
Les racines carrées de 24+10i sont donc 5+i et -5-i

Les racines carrées de -24-10i
Posons z=x+iy \text{ / } z^2=-24-10i
On a : z^2=-24-10i=i^2(24+10i)\Longrightarrow \left(\dfrac{z}{i}\right)^2=24+10i
On en déduit directement que i(-5-i)=1-5i \text{ et }i(5+i)=-1+5i sont les racines carrées de -24-10i

Conclusion :
Les racines quatrièmes de 476+480i sont : \boxed{-5-i\enskip , \enskip 5+i\enskip , \enskip-1+5i\enskip , \enskip 1-5i}

exercice 6


1) Les expressions de \cos 5\theta \enskip \text{ et }\enskip \sin 5\theta

On a : \cos 5\theta+i\sin 5\theta=(\cos\theta+i\sin\theta)^5=\cos^5\theta+5i\cos^4\theta\sin\theta-10\cos^3\theta\sin^2\theta-10i\cos^2\theta\sin^3\theta+5\cos\theta\sin^4\theta+i\sin^5\theta
En identifiant les parties réelle et imaginaire : \cos 5\theta=\cos^5\theta-10\cos^3\theta\sin^2\theta+5\cos\theta\sin^4\theta\enskip \text{ et } \enskip \sin 5\theta= 5\cos^4\theta\sin\theta-10\cos^2\theta\sin^3\theta+\sin^5\theta

Or : \cos^2\theta=1-\sin^2\theta \enskip \text{ et }\enskip \sin^2\theta=1-\cos^2\theta

On conclut :
\cos 5\theta=\cos^5\theta-10\cos^3\theta(1-\cos^2\theta)+5\cos\theta (1-\cos^2\theta)^2=\boxed{16\cos^5\theta-20\cos^3\theta+5\cos\theta} \\\\ \sin 5\theta= 5(1-\sin^2\theta)^2\sin\theta-10(1-\sin^2\theta)\sin^3\theta+\sin^5\theta=\boxed{16\sin^5\theta-20\sin^3\theta+5\sin\theta}


2) Pour \theta\in\R\backslash\left\lbrace(2k+1)\dfrac{\pi}{10}\enskip \text{ avec } k\in\Z\right\rbrace

\begin{matrix} \tan 5\theta &=& \dfrac{\sin 5\theta}{\cos 5\theta} &=& \dfrac{16\sin^5\theta-20\sin^3\theta+5\sin\theta}{16\cos^5\theta-20\cos^3\theta+5\cos\theta} \\\\&=&\dfrac{\dfrac{16\sin^5\theta-20\sin^3\theta+5\sin\theta}{\cos^{5}\theta}}{\dfrac{16\cos^5\theta-20\cos^3\theta+5\cos\theta}{\cos^{5}\theta}}&=& \dfrac{16\tan^5\theta-20\dfrac{\tan^3\theta}{\cos^2\theta}+5\dfrac{\tan\theta}{\cos^4\theta}}{16-\dfrac{20}{\cos^2\theta}+\dfrac{5}{\cos^4\theta}} \\\\&=&\dfrac{16\tan^5\theta-20\tan^3\theta(1+\tan^2\theta)+5\tan\theta(1+\tan^2\theta)^2}{16-20(1+\tan^2\theta)+5(1+\tan^2\theta)^2}&=&\boxed{\dfrac{\tan^5\theta-10\tan^3\theta+5\tan\theta}{5\tan^4\theta-10\tan^2\theta+1}}\end{matrix}

exercice 7


Pour x\in\R\text{ : }

\begin{array}{cl}\cos^7 x &= \left(\dfrac{\text{e}^{ix}+\text{e}^{-ix}}{2}\right)^7  \\\\&= \dfrac{1}{128}\left(\text{e}^{ix}+\text{e}^{-ix}\right)^7 \\\\&=\dfrac{1}{128}\displaystyle\sum_{k=0}^{7} {7\choose k}\text{e}^{ikx}\text{e}^{-i(7-k)x} \\\\&= \dfrac{1}{128}\left(\text{e}^{-i7x}+7\text{e}^{ix}\text{e}^{-i6x}+21\text{e}^{i2x}\text{e}^{-i5x}+35\text{e}^{i3x}\text{e}^{-i4x}+35\text{e}^{i4x}\text{e}^{-i3x}+21\text{e}^{i5x}\text{e}^{-i2x}+7\text{e}^{i6x}\text{e}^{-ix}+\text{e}^{i7x}) \\\\&=\dfrac{1}{64}\left[\left(\dfrac{\text{e}^{i7x}+\text{e}^{-i7x}}{2}\right)+7\left(\dfrac{\text{e}^{i5x}+\text{e}^{-i5x}}{2}\right)+21\left(\dfrac{\text{e}^{i3x}+\text{e}^{-i3x}}{2}\right)+35\left(\dfrac{\text{e}^{ix}+\text{e}^{-ix}}{2}\right)\right] \\\\&=\boxed{\dfrac{1}{64}(\cos 7x +7\cos 5x +21\cos 3x +35\cos x)}\end{array}

exercice 8

1) Soit p\in\N. D'après les formules d'Euler, on a : \cos^{2p+1} x =\left(\dfrac{\text{e}^{ix}+\text{e}^{-ix}}{2}\right)^{2p+1}
D'où, en utilisant la formule du binôme de Newton : \cos^{2p+1} x =\dfrac{1}{2^{2p+1}}\displaystyle \sum_{k=0}^{2p+1}{2p+1\choose k } \text{e}^{i(2p+1-2k)x}
On décompose en deux sommes comme suit : \cos^{2p+1} x =\dfrac{1}{2^{2p+1}}\left[\displaystyle \sum_{k=0}^{p}{2p+1\choose k } \text{e}^{i(2p+1-2k)x}+\displaystyle \sum_{k=p+1}^{2p+1}{2p+1\choose k } \text{e}^{i(2p+1-2k)x}\right]
Effectuons le changement d'indice t=2p+1-k dans la dernière somme :

\displaystyle \sum_{k=p+1}^{2p+1}{2p+1\choose k } \text{e}^{i(2p+1-2k)x}=\sum_{t=0}^{p}{2p+1\choose 2p+1-t } \text{e}^{i(2t-(2p+1))x}=\sum_{t=0}^{p}{2p+1\choose t } \text{e}^{i(2t-(2p+1))x}\enskip \enskip \left(\text{ car : }{2p+1\choose 2p+1-t }={2p+1\choose t }\right)

L'indice t étant muet, on peut le remplacer par k, ainsi, en regroupant les deux sommes :

\begin{array}{cl}\cos^{2p+1}x &=\dfrac{1}{2^{2p+1}}\displaystyle \sum_{k=0}^{p}{2p+1\choose k } \left(\text{e}^{i(2p+1-2k)x}+\text{e}^{-i(2p+1-2k)x}\right)\\\\&=\boxed{\dfrac{1}{2^{2p}}\displaystyle \sum_{k=0}^{p}{2p+1\choose k } \cos\left((2p+1-2k)x\right)}\end{array}

2) Soit p\in\N. D'après les formules d'Euler, on a : \sin^{2p+1} x =\left(\dfrac{\text{e}^{ix}-\text{e}^{-ix}}{2}\right)^{2p+1}
D'où, en utilisant la formule du binôme de Newton : \sin^{2p+1} x =\dfrac{1}{(2i)^{2p+1}}\displaystyle \sum_{k=0}^{2p+1}{2p+1\choose k } (-1)^{k}\text{e}^{i(2p+1-2k)x}
On décompose en deux sommes comme suit : \sin^{2p+1} x =\dfrac{1}{(2i)^{2p+1}}\left[\displaystyle \sum_{k=0}^{p}{2p+1\choose k } (-1)^k\text{e}^{i(2p+1-2k)x}+\displaystyle \sum_{k=p+1}^{2p+1}{2p+1\choose k } (-1)^k\text{e}^{i(2p+1-2k)x}\right]
Effectuons le changement d'indice t=2p+1-k dans la dernière somme :

\displaystyle \sum_{k=p+1}^{2p+1}{2p+1\choose k }(-1)^k \text{e}^{i(2p+1-2k)x}=\sum_{t=0}^{p}{2p+1\choose 2p+1-t }(-1)^{2p+1-t} \text{e}^{-i(2p+1-2t)x}=\sum_{t=0}^{p}{2p+1\choose t } (-1)^{t+1}\text{e}^{-i(2p+1-2t)x}\enskip

\enskip \left(\text{ car : }{2p+1\choose 2p+1-t }={2p+1\choose t }\enskip \text{ et } (-1)^{2p+1-t}=(-1)^{t+1}\right)
L'indice t étant muet, on peut le remplacer par k, ainsi, en regroupant les deux sommes :

\begin{array}{cl}\sin^{2p+1}x &=\dfrac{1}{(2i)^{2p+1}}\displaystyle \sum_{k=0}^{p}{2p+1\choose k }(-1)^k \left(\text{e}^{i(2p+1-2k)x}-\text{e}^{-i(2p+1-2k)x}\right)\\\\&=\boxed{\dfrac{(-1)^p}{2^{2p}}\displaystyle \sum_{k=0}^{p}(-1)^k{2p+1\choose k } \sin\left((2p+1-2k)x\right)}\end{array}

Car : \dfrac{2i}{(2i)^{2p+1}}=\dfrac{1}{(2i)^{2p}}=\dfrac{1}{2^{2p}\enskip i^{2p}}=\dfrac{1}{2^{2p}(-1)^p}=\dfrac{(-1)^p}{2^{2p}}

exercice 9

1) Le discriminant est \Delta=(-16)^2-4\times 4(11-12i)=16(5+12i)=16(9+2\times 6i-4=(3+2i)^2
Les racines carrées de \Delta sont donc 12+8i et -12-8i
L'équation possède donc deux solutions : z_1=\dfrac{16+12+8i}{8}=\dfrac{7}{2}+i \enskip \text{ et }\enskip z_2=\dfrac{16-12-8i}{8}=\dfrac{1}{2}-i

\boxed{S=\left\lbrace \dfrac{7}{2}+i\enskip , \enskip \dfrac{1}{2}-i\right\rbrace}

2) Le discriminant est \Delta = (1+4i)^2-4\times 1 (5+5i)=-35-12i
Posons x+iy \text{ / } (x+iy)^2=-35-12i

\begin{cases} x^2+y^2=|\Delta|=37 \\ x^2-y^2=\mathcal{R}e(\Delta)=-35 \\ 2ab=\mathcal{I}m(\Delta)=-12\end{cases}

On en tire les racines du discriminant \Delta \enskip \text{ : }\enskip 1-6i \text{ et } -1+6i
L'équation possède donc deux solutions : z_1=\dfrac{1+4i+1-6i}{2}=1-i \enskip \text{ et }\enskip z_2=\dfrac{1+4i-1+6i}{2}=5i

\boxed{S=\left\lbrace 1-i \enskip , \enskip 5i\right\rbrace}

3) On pose Z=z^4, on obtient l'équation Z^2+4Z+16=0 de discriminant \Delta=4^2-4\times 16=-3\times 16 = (4i\sqrt{3})^2
L'équation en Z possède donc deux solutions complexes conjuguées :

Z_1=\dfrac{-4+4i\sqrt{3}}{2}=-2+2i\sqrt{3} \text{ et } Z_2=-2-2i\sqrt{3}
On cherche alors les solutions des deux équations : z^4=Z_1 \text{ et }z^4=Z_2.

Z_1=4\left(-\dfrac{1}{2}+i\dfrac{\sqrt{3}}{2}\right)=4\text{e}^{\frac{2i\pi}{3}}=\left(\sqrt{2}\text{e}^{i\frac{\pi}{6}}\right)^4

Les racines quatrièmes de 1 étant 1,i,-1\text{ et }-i, on en déduit que : z^4=Z_1 \iff z\in\left\lbrace \sqrt{2}\text{e}^{i\frac{\pi}{6}}\text{ , }i\sqrt{2}\text{e}^{i\frac{\pi}{6}}\text{ , }-\sqrt{2}\text{e}^{i\frac{\pi}{6}}\text{ , }-i\sqrt{2}\text{e}^{i\frac{\pi}{6}}\right\rbrace

De plus, Z_2=\bar{Z_1}=4\text{e}^{-\frac{2i\pi}{3}}=\left(\sqrt{2}\text{e}^{-i\frac{\pi}{6}}\right)^4

On en déduit que : z^4=Z_2 \iff z\in\left\lbrace \sqrt{2}\text{e}^{i\frac{-\pi}{6}}\text{ , }i\sqrt{2}\text{e}^{-i\frac{\pi}{6}}\text{ , }-\sqrt{2}\text{e}^{-i\frac{\pi}{6}}\text{ , }-i\sqrt{2}\text{e}^{-i\frac{\pi}{6}}\right\rbrace

L'équation en z possède alors huit solutions :

\boxed{S=\left\lbrace \sqrt{2}\text{e}^{i\frac{\pi}{6}}\text{ , }\sqrt{2}\text{e}^{\frac{2i\pi}{3}}\text{ , }\sqrt{2}\text{e}^{\frac{7i\pi}{6}}\text{ , }\sqrt{2}\text{e}^{-i\frac{\pi}{3}} \text{ , }\sqrt{2}\text{e}^{-i\frac{\pi}{6}}\text{ , }\sqrt{2}\text{e}^{\frac{i\pi}{3}}\text{ , }\sqrt{2}\text{e}^{\frac{5i\pi}{6}}\text{ , }\sqrt{2}\text{e}^{\frac{-2i\pi}{3}} \right\rbrace}


4- On factorise (z^2+z+1)^2+1=(z^2+z+1)^2-(i)^2=(z^2+z+1-i)(z^2+z+1-i)
Il suffit donc de déterminer les racines de chacun des deux polynômes.
Le discriminant du premier est \Delta=-3+4i=(1+2i)^2, ses racines sont donc: z_1=\dfrac{-1-(1+2i)}{2}=-1-i \enskip \text{ et } z_2=\dfrac{-1+(1+2i)}{2}=i
De même, le discriminant du second \Delta'=-3-4i=(1-2i)^2, on en déduit ses racines : z_3=\dfrac{-1-(1-2i)}{2}=-1+i \enskip \text{ et } z_4=\dfrac{-1+(1-2i)}{2}=-i

Conclusion : \boxed{S=\left\lbrace -1-i \enskip , \enskip i\enskip , \enskip-1+i\enskip , \enskip -i\right\rbrace}

exercice 10

On a : S+iS'=\displaystyle\sum_{k=0}^{n} {n\choose k} (\cos (kx) + i \sin(kx))=\sum_{k=0}^{n} {n\choose k}\text{e}^{ikx}=(\text{e}^{ix}+1)^n

En factorisant par l'angle moitié : S+iS'=(\text{e}^{ix}+1)^n=\left[\text{e}^{i\frac{x}{2}}(\text{e}^{i\frac{x}{2}}+\text{e}^{-i\frac{x}{2}})\right]^n=2^n\text{e}^{i\frac{xn}{2}}\left(\dfrac{\text{e}^{i\frac{x}{2}}+\text{e}^{-i\frac{x}{2}}}{2}\right)^n=2^n\text{e}^{i\frac{xn}{2}}\cos^n\left(\dfrac{x}{2}\right)

On en déduit :
\boxed{\begin{matrix}S=\mathcal{R}e(S+iS')=2^n\cos^n\left(\dfrac{x}{2}\right)\cos\left(\dfrac{nx}{2}\right) \\ S'=\mathcal{I}m(S+iS')=2^n\cos^n\left(\dfrac{x}{2}\right)\sin\left(\dfrac{nx}{2}\right)\end{matrix}}


exercice 11

On a :

\begin{array}{cl}AB &= \left(z+z^2+z^4\right)(z^3+z^5+z^6)  \\\\&= z^4+z^6+z^7+z^5+z^7+z^8+z^7+z^9+z^{10} \\\\&=z^4(1+z+z^2+z^3+z^4+z^5+z^6)+2z^7 \\\\&=z^4\times 0+2\text{e}^{i2\pi} \\\\&=\boxed{2}\end{array}

Et :

\begin{array}{cl}A+B &= z+z^2+z^3+z^4+z^5+z^6 \\\\&= 1+z+z^2+z^3+z^4+z^5+z^6-1 \\\\&=\boxed{-1}\end{array}

Puisque AB=2 \text{ et }A+B=-1 \text{ , donc }A \text{ et }B \text{ sont solutions de l'équation } X^2+X+2=0

Les solutions de cette équation sont : -\dfrac{1}{2}-i\dfrac{\sqrt{7}}{2} \enskip \text{ et } \enskip -\dfrac{1}{2}+i\dfrac{\sqrt{7}}{2}
Il faut étudier les parties imaginaires pour déterminer la solution qui correspond à A et celle qui correspond à B

On a A=z+z^2+z^4 \Longrightarrow \mathcal{I}m(A)=\sin\left(\dfrac{2\pi}{7}\right)+\sin\left(\dfrac{4\pi}{7}\right)+\sin\left(\dfrac{8\pi}{7}\right)

Puisque : \dfrac{1}{2}<\sin\left(\dfrac{2\pi}{7}\right)<1\enskip ; \enskip \dfrac{1}{2}<\sin\left(\dfrac{4\pi}{7}\right)<1\enskip ; \enskip -\dfrac{1}{2}<\sin\left(\dfrac{8\pi}{7}\right)<0\enskip ; \enskip\text{ Alors : }0< \mathcal{I}m(A)

On en déduit que : \boxed{ A=-\dfrac{1}{2}+i\dfrac{\sqrt{7}}{2} \enskip \text{ et } \enskip B=-\dfrac{1}{2}-i\dfrac{\sqrt{7}}{2}}

exercice 12

1) Fait en exercice 6 : \cos 5x=16\cos^5 x-20\cos^3 x +5\cos x = \boxed{\cos x( 16\cos^4 x-20\cos^2 x +5)}

2) Si x=\dfrac{\pi}{10}

\cos\dfrac{\pi}{10}\left(16\cos^4 \dfrac{\pi}{10}-20\cos^2 \dfrac{\pi}{10} +5\right)=\cos\dfrac{5\pi}{10}=\cos\dfrac{\pi}{2}\iff \cos\dfrac{\pi}{10}\left(16\cos^4 \dfrac{\pi}{10}-20\cos^2 \dfrac{\pi}{10} +5\right)=0

Et puisque \cos\dfrac{\pi}{10}\neq 0 , alors :
\boxed{ \cos\dfrac{\pi}{10} \text{ est racine du polynôme } 16X^4-20X^2+5}


3) Notons t=X^2 et cherchons les racines de 16t^2-20t+5

Le discriminant est \Delta=(-20)^2-4\times 5\times 16=80=(4\sqrt{5})^2
Les racines carrées sont donc t_1=\dfrac{20-4\sqrt{5}}{32}=\dfrac{5-\sqrt{5}}{8} \text{ et }t_2=\dfrac{20+4\sqrt{5}}{32}=\dfrac{5+\sqrt{5}}{8}
Or, \cos^2 \dfrac{\pi}{10} \geq \cos^2\dfrac{\pi}{6} =\dfrac{3}{4}\enskip \text{ et } \enskip t_1=\dfrac{5-\sqrt{5}}{8}\approx 0,35 \text{ et }t_2=\dfrac{5+\sqrt{5}}{8}\approx 0,9

On en déduit que : \boxed{\cos^2\dfrac{\pi}{10}=\dfrac{5+\sqrt{5}}{8}}

Enfin, on a pour x\in\R \text{ : } \cos(2x)=2\cos^2 x-1

Donc : \cos\dfrac{\pi}{5}=2\cos^2\dfrac{\pi}{10}-1=\dfrac{5+\sqrt{5}}{4}-1\iff \boxed{\cos\dfrac{\pi}{5}=\dfrac{\sqrt{5}+1}{4}}

exercice 13

1) \text{ Soient } (z,z')\in\C^2 \text{ .  On a : } \enskip |z+z'|^2+|z-z'|^2=(z+z')\overline{(z+z')}+(z-z')\overline{(z-z')}=(z+z')(\bar{z}+\bar{z'})+(z-z')(\bar{z}-\bar{z'})=2z\bar{z}+2z'\bar{z'}=\boxed{2(|z|^2+|z'|^2)}

2) \text{ Soit } \omega\in\C \text{ tel que } \omega^2=zz' , on a :

\begin{array}{cl}\left(\left|\omega+\dfrac{z+z'}{2}\right|+\left|\omega-\dfrac{z+z'}{2}\right|\right)^2 &=  \left|\omega+\dfrac{z+z'}{2}\right|^2+\left|\omega-\dfrac{z+z'}{2}\right|^2+2\left|\omega^2-\left(\dfrac{z+z'}{2}\right)^2\right| \\\\&=2|\omega|^2+2\dfrac{|z+z'|^2}{4}+2\left|\dfrac{4\omega^2-z^2-2zz'-z'^2}{4}\right| \\\\&=2|zz'|+\dfrac{|z+z'|^2}{2}+\dfrac{\left|4zz'-z^2-2zz'-z'^2\right|}{2} \\\\&= 2|zz'|+\dfrac{|z+z'|^2}{2}+\dfrac{|z-z'|^2}{2}  \\\\&=2|zz'|+\dfrac{1}{2}\left(|z-z'|^2+|z+z'|^2\right)  \\\\&=\boxed{(|z|+|z'|)^2}\end{array}

exercice 14

Le complexe nul n'ayant pas d'argument, on considère que z\not=0 et z\not=-i

Pour z\in\C^*\backslash\lbrace -i\rbrace

\begin{array}{cl}2\arg(z+i)\equiv \text{arg}(z)+\text{arg} (i) \text{ } [2\pi}] &\Longrightarrow  \text{ arg}(z+i)^2\equiv \text{arg} (iz) \text{ } [2\pi}]  \\\\&\Longrightarrow  \dfrac{(z+i)^2}{iz}=\overline{\left(\dfrac{ (z+i)^2}{iz}\right)}  \\\\&\Longrightarrow  \dfrac{(z+i)^2}{iz}=\dfrac{(\overline{z}-i)^2}{-i\overline{z}}   \\\\&\Longrightarrow -\overline{z}(z+i)^2=z(\overline{z}-i)^2     \\\\&\Longrightarrow -\overline{z}(z^2+2iz-1)=z(\overline{z}^2-2i\overline{z}-1)  \\\\&\Longrightarrow -z\overline{z}(z+\overline{z})+z+\overline{z}=0    \\\\&\Longrightarrow (z+\overline{z})(1-|z|^2)=0  \\\\&\Longrightarrow z=-\overline{z}\text{ ou }|z|=1\end{array}

Or, on sait que i\R=\left\lbrace z\in\C\text{ / }z=-\overline{z}\right\rbrace \text{ et } \U=\left\lbrace z\in\C\text{ / }|z|=1\right\rbrace

Donc l'ensemble des solutions est à première vue : \displaystyle S=\left(i\R\cup \U\right)\backslash \lbrace 0,-i\rbrace = \left(i\R^*\cup\U\right)\backslash\lbrace -i\rbrace

La réciproque :

\bullet \text{ Soit } z\in i\mathbb{R}^{*} \backslash\lbrace -i\rbrace \text{ , alors } z=iy \text{ avec } y\in\mathbb{R}^{*}\backslash \lbrace{-1\rbrace}

On a : \dfrac{(z+i)^2} {iz} = \dfrac{(iy+i)^2} {-y} = \dfrac{(y+1)^2} {y}
Alors \dfrac{(z+i)^2} {iz}\in\mathbb{R}^{+} \text{ si et seulement si }  y\in\mathbb{R}^{*+}

\bullet Soit z\in \mathbb{U}, alors z=\text{e}^{i\theta} \text{ tel que } \theta\in\mathbb{R}

On a:
\begin{array}{cl} \dfrac{(z+i)^2} {iz} &= \dfrac{(\text{e}^{i\theta}+i)^2} {i\text{e}^{i\theta}}  \\\\&= \dfrac{\text{e}^{2i\theta}+2i\text{e}^{i\theta}-1} {i\text{e}^{i\theta}}   \\\\&=-i\text{e}^{i\theta}+2+i\text{e}^{-i\theta}   \\\\&= 2-i(\text{e}^{i\theta}-\text{e}^{-i\theta})  \\\\&=2+2\left(\dfrac{\text{e}^{i\theta}-\text{e}^{-i\theta}}{2i}\right)  \\\\&=2(1+\sin \theta)  \\\\&\geq 0 \text{ ( En effet, } \forall \theta \in\mathbb{R} \text{ : } \sin\theta \geq -1 \text{ )}\end{array}

On conclut que : \boxed{ S= \left(i\mathbb{R}^{*+} \cup \mathbb{U}\right)\backslash\lbrace{-i\rbrace} }

exercice 15

On avait vu dans le cours que \forall z,z'\in\C \text{ : } |z+z'|=|z|+|z'| \text{ si et seulement si } z \text{ ou }z'  \text{ est nul, ou si } \exists a\in\R^{+} \text{ / }z=az'

Montrons que cette propriété reste valable dans le cas général, c'est-à-dire, montrons que :
\displaystyle\sum_{k=1}^{n}|z_k|=\left|\sum_{k=1}^{n}z_k\right| \enskip  \text{ si et seulement si } \forall k\in\lbrace 1,\cdots, n\rbrace \text{ : }\exists a_k\in\R^+ \text{ / } z_k=a_kz_1

Par récurrence :
Pour n=2 , vrai d'après le cours.
Soit la propriété vraie pour un entier n\geq 1 , et on choisit (z_1,\codts, z_n,z_{n+1})\in\C^{n+1} \text{ / } \displaystyle\sum_{k=1}^{n+1}|z_k|=\left|\sum_{k=1}^{n+1}z_k\right|

On a :
\displaystyle\left|\sum_{k=1}^{n+1}z_k\right|=\left|\sum_{k=1}^{n}z_k+z_{n+1}\right|\leq \left|\sum_{k=1}^{n}z_k\right|+|z_{n+1}|\leq  \sum_{k=1}^{n}|z_k|+|z_{n+1}|=\underbrace{\sum_{k=1}^{n+1}|z_k|=\left|\sum_{k=1}^{n+1}z_k\right|}_{\text{D'après l'hypothèse}}

Il s'ensuit que toutes ces expressions sont égales et en particulier :

\displaystyle \left|\sum_{k=1}^{n}z_k\right|+|z_{n+1}|=\sum_{k=1}^{n}\left|z_k\right|+|z_{n+1}| \enskip \text{ (I)} \\ \left|\sum_{k=1}^{n}z_k+z_{n+1}\right|=\sum_{k=1}^{n}\left|z_k\right|+|z_{n+1}|\enskip \text{ (II)}

\bullet De \text{ (I)  on tire : } \displaystyle \left|\sum_{k=1}^{n}z_k\right|=\sum_{k=1}^{n}\left|z_k\right|
Il existe donc d'après l'hypothèse (a_1,\cdots,a_n)\in\R^{n}_{+} \text{ / } \forall p\in  \lbrace{1,\cdots, n\rbrace \text{ : } z_p=a_pz_1 \text{ (i) }

\bullet De \text{ (II) }, on déduit : \exists a\in\R^+ \text{ / } z_{n+1}=a\displaystyle \left(\sum_{k=1}^{n}z_k\right)\enskip  \text{ (ii) }

De \text{ (i) et (ii) } : \displaystyle z_{n+1}=a \left(\sum_{k=1}^{n}z_k\right)=a\left(\sum_{p=1}^{n}a_p z_p\right)=\left(\sum_{p=1}^{n}a_p a\right) z_1

Notons a_{n+1}=\displaystyle \sum_{p=1}^{n}a_p a\in\R_{+}

On a donc : \forall p\in\lbrace 1,\cdots, n, n+1\rbrace \text{ : }\exists a_p\in\R_{+} \text{ / }z_p=a_pz_1

L'hypothèse est donc vraie pour n+1

Conclusion : \boxed{\displaystyle\sum_{k=1}^{n}|z_k|=\left|\sum_{k=1}^{n}z_k\right| \enskip  \text{ si et seulement si } \forall k\in\lbrace 1,\cdots, n\rbrace \text{ : }\exists a_k\in\R^+ \text{ / } z_k=a_kz_1 }

exercice 16

1) On a :

\begin{array}{cl} z^2-2z+7\in\R &\iff \overline{z^2-2z+7}=z^2-2z+7  \\\\&\iff \bar{z}^2-2\bar{z}+7=z^2-2z+7  \\\\&\iff (z^2-\bar{z}^2)-2(z-\bar{z})=0  \\\\&\iff (z-\bar{z})(z+\bar{z}+2)=0  \\\\&\iff (z-\bar{z})(2\mathcal{R}e(z)-2)=0  \\\\&\iff z=\bar{z} \text{ ou } \mathcal{R}e(z)=1  \\\\&\iff z\in\R\text{ ou } \mathcal{R}e(z)=1\end{array}

\boxed{ \text{L'ensemble cherché est donc constitué de l'axe réel et de la droite d'équation }x=1}

2)

\left|\dfrac{z-i}{z+2}\right|=1\iff |z-i|=|z+2|\iff MJ=MP
J et P sont les points d'affixes respectives i et -2

\boxed{ \text{L'ensemble cherché est donc la médiatrice du segment }[PJ] }

3)

z^2-(1+i)^2=\bar{z}^2-(1-i)^2 \iff z^2-\bar{z}^2=(1+i)^2-(1-i)^2\iff (z+\bar{z})(z-\bar{z})=4i
En posant z=x+iy \text{( avec } x,y\in\R ) , on obtient : 2x\times 2iy =4i \iff xy=1

\boxed{ \text{L'ensemble cherché est donc l'hyperbole d'équation }y=\dfrac{1}{x} }

4)

Notons O l'origine du plan complexe et I le point d'affixe 1. On a :

|z|=|z-1|\iff OM=IM \iff M \text{ appartient à la médiatrice de } [OI]

|z|=\left|\dfrac{1}{z}\right|\iff|z|=1\iff M \text{ appartient au cercle trigonométrique } \U

\boxed{ \text{L'ensemble cherché est donc les deux points d'affixes }\text{e}^{i\frac{\pi}{3}}\text{ et } \text{e}^{-i\frac{\pi}{3}}}

exercice 17

Posons a=\text{e}^{i\theta} .

Soit k\in\lbrace 0,\cdots, n-1\rbrace , on a z_k=\exp\left(i\dfrac{2k\pi+\theta}{n}\right). En effet :

z_k^n=&\iff \left(z_k \text{e}^{-i\frac{\theta}{n}}\right)^n=1\iff z_k\text{e}^{-i\frac{\theta}{n}}=\text{e}^{i\frac{2k\pi}{n}}\iff z_k=\exp\left(i\dfrac{2k\pi+\theta}{n}\right)

Donc : 1+z_k=1+\exp\left(i\dfrac{2k\pi+\theta}{n}\right)=\exp\left(i\dfrac{2k\pi+\theta}{2n}\right)\left( \exp\left(-i\dfrac{2k\pi+\theta}{2n}\right) +\exp\left(i\dfrac{2k\pi+\theta}{2n}\right)   \right)=2\cos\left(\dfrac{2k\pi+\theta}{2n}\right) \exp\left(i\dfrac{2k\pi+\theta}{2n}\right)

Il s'ensuit :

(1+z_k)^n=2^n\cos^n\left(\dfrac{2k\pi+\theta}{2n}\right)\text{e}^{ik\pi} \text{e}^{i\frac{\theta}{2}}=2^n(-1)^k\cos^n\left(\dfrac{2k\pi+\theta}{2n}\right) \text{e}^{i\frac{\theta}{2}}

Tous les points d'affixes (1+z_k)^n sont donc situés sur la droite qui fait un angle \dfrac{\theta}{2} avec l'axe des abscisses.

Conclusion : \boxed{\text{ Les points d'affixes } (1+z_1)^n,\cdots,(1+z_n)^n \text{ sont tous alignés }}

exercice 18

\red 1)\iff 2)

\begin{array}{cl} ABC \text{ triangle équilatéral direct ou indirect}  &\iff AB=AC \text{ et } \left((\overrightarrow{AB},\overrightarrow{AC})\equiv \dfrac{\pi}{3} \enskip [2\pi]\text{ ou } (\overrightarrow{AB},\overrightarrow{AC})\equiv -\dfrac{\pi}{3} \enskip [2\pi]\right) \\\\&\iff |c-a|=|b-a|\text{ et }\left(\arg\left(\dfrac{c-a}{b-a}\right)\equiv \dfrac{\pi}{3} \enskip [2\pi]\text{ ou } \arg\left(\dfrac{c-a}{b-a}\right)\equiv -\dfrac{\pi}{3} \enskip [2\pi]\right) \\\\&\iff \left|\dfrac{c-a}{b-a}\right|=1\text{ et }\left(\arg\left(\dfrac{c-a}{b-a}\right)\equiv \dfrac{\pi}{3} \enskip [2\pi]\text{ ou } \arg\left(\dfrac{c-a}{b-a}\right)\equiv -\dfrac{\pi}{3} \enskip [2\pi]\right) \\\\&\iff \dfrac{c-a}{b-a}=\text{e}^{i\frac{\pi}{3}} \text{ ou } \dfrac{c-a}{b-a}=\text{e}^{-i\frac{\pi}{3}}  \\\\&\iff \dfrac{c-a}{b-a}=-j^2 \text{ ou } \dfrac{c-a}{b-a}=-j  \\\\&\iff c-a =-j^2b+j^2 a \text{ ou } c-a = -jb+ja  \end{array} \begin{array}{cl}\white ABC  \text{ triangle équilatéral direct ou indirect}  \black \\\\&\iff a(-j^2-1)+j^2b+c=0\text{ ou } a(-j-1)+jb+c=0 \\\\&\iff aj+bj^2+c=0 \text{ ou } aj^2+bj+c=0  \\\\&\iff aj^4+bj^2+c=0 \text{ ou } aj^2+bj+c=0  \\\\&\iff \boxed{ j \text{ ou }j^2\text{ est racine du polynôme }aX^2+bX+c}  \end{array}

\red 2)\iff 3)

\begin{array}{cl} j \text{ ou }j^2\text{ est racine du polynôme }aX^2+bX+c  &\iff (aj^4+bj^2+c)(aj^2+bj+c)=0 \\\\&\iff a^2j^6+abj^5+acj^4+abj^4+b^2j^3+bcj^2+acj^2+bcj+c^2=0 \\\\&\iff a^2+abj^2+acj+abj+b^2+bcj^2+acj^2+bcj+c^2=0 \\\\&\iff a^2+ab(j+j^2)+ac(j+j^2)+bc(j+j^2)+b^2+c^2=0 \\\\&\iff a^2+b^2+c^2-ab-ac-bc=0 \\\\&\iff \boxed{a^2+b^2+c^2=ab+ac+bc} \end{array}

\red 3)\iff 4)

\begin{array}{cl} a^2+b^2+c^2=ab+ac+bc  &\iff 2a^2+2b^2+2c^2=2ab+2ac+2bc \\\\&\iff 2a^2+2b^2+2c^2-2ab-2ac-2bc=0 \\\\&\iff a^2-2ab+b^2+a^2-2ac+c^2+b^2-2bc+c^2=0 \\\\&\iff \boxed{(b-a)^2+(c-b)^2+(a-c)^2=0} \end{array}

exercice 19

Soit M d'affixe z, on note M_1(z_1) l'image de M par r_1 et M'(z') l'image de M_1 par r_2. On a alors :

z_1+1=\text{e}^{i\frac{\pi}{3}}(z+1)=-j^2(z+1)\\z'-j=\text{e}^{i\frac{2\pi}{3}}(z_1-j)=j(z_1-j)

On en déduit :

\begin{array}{cl} z' &=j+j(-1-j^2(z+1)-j)	 \\\\&=j+j(j^2-j^2(z+1)) \\\\&=j+1-(z+1) \\\\&=-z+j \end{array}

On reconnaît la rotation d'angle \arg(-1)\equiv \pi\enskip [2\pi] et de centre \Omega d'affixe j^2
Or, on sait qu'une rotation d'angle \pi est une symétrie centrale.

Conclusion : \boxed{ r_2\circ r_1 \text{ est une symétrie centrale de centre }\Omega(j^2)}

exercice 20

Puisqu'on a invariance par translation et par rotation de centre le centre du cercle, on peut supposer que le cercle est de centre O et que les sommets du polygone sont les images des nombres complexes R\exp\left(\frac{i2k\pi}{n}\right) \enskip , \enskip k\in\lbrace 1,2,\cdots, n\rbrace

Pour tout point M du cercle, d'affixe z et donc de module R, on a :

\begin{array}{cl} f(M)  &=\displaystyle \sum_{k=1}^{n} MA_k^2 \\\\&=\displaystyle \sum_{k=1}^{n}\left|R\exp\left(\frac{i2k\pi}{n}\right)-z\right|^2 \\\\&=\displaystyle \sum_{k=1}^{n}\left(R\exp\left(\frac{i2k\pi}{n}\right)-z\right)\left(R\exp\left(\frac{i2k\pi}{n}\right)-\bar{z}\right) \\\\&=\displaystyle \sum_{k=1}^{n}\left(R)2-Rz\exp\left(\frac{i2k\pi}{n}\right)-R\bar{z}\exp\left(\frac{i2k\pi}{n}\right)+R^2\right) \\\\&=\displaystyle 2nR^2-2R \left[\mathcal{R}e\left(\bar{z}\sum_{k=1}^{n}\exp\left(\frac{i2k\pi}{n}\right)\right)\right] \end{array}

Or, la somme des racines n-ièmes de l'unité est nulle, donc : \displaystyle\sum_{k=1}^{n}\exp\left(\frac{i2k\pi}{n}\right)=0

On en déduit que : \boxed{f(M)=2nR^2 \text{ , qui est bien indépendant du choix du point M sur le cercle }}
Publié le
ceci n'est qu'un extrait
Pour visualiser la totalité des cours vous devez vous inscrire / connecter (GRATUIT)
Inscription Gratuite se connecter
Merci à
Panter Correcteur
pour avoir contribué à l'élaboration de cette fiche


Vous devez être membre accéder à ce service...

Pas encore inscrit ?

1 compte par personne, multi-compte interdit !

Ou identifiez-vous :


Rester sur la page

Inscription gratuite

Fiches en rapport

parmi 1675 fiches de maths

Désolé, votre version d'Internet Explorer est plus que périmée ! Merci de le mettre à jour ou de télécharger Firefox ou Google Chrome pour utiliser le site. Votre ordinateur vous remerciera !